11
$\begingroup$

Let $\square_2=\{(x,y): 0\leq x, y\leq1\}$ be the unit square in $\mathbb{R}^2$. Take $n>1$ points $P_1, \dots, P_n\in\square_2$.

Denote the distances $d_j=\min\{\Vert P_k-P_j\Vert: k\neq j\}$, for $j=1,\dots, n$.

QUESTION. Is it true that $d_1^2+\cdots+d_n^2\leq 4$? The bound is tight when $n=2$.

$\endgroup$
3
  • $\begingroup$ ...and tight when $n= 4$ (points at corners). $\endgroup$ Jan 16, 2019 at 19:43
  • $\begingroup$ A close result: any $n$ points in a right triangle $ABC$, $\angle ABC=\pi/2$, may be enumerated by $P_1,\dots,P_n$ so that $AP_1^2+P_1P_2^2+\dots+P_{n-1}P_n^2+P_nC^2\leqslant AB^2$. The proof is similar to Iosif Pinelis's idea: take a height $BH$ and apply the induction proposition to triangles $AHB,CHB$ (if all point lie in the same triangle, proceed partitioning.) $\endgroup$ Jan 16, 2019 at 20:17
  • $\begingroup$ Perhaps this answer by @achillehui will help: Average distance between $n$ randomly distributed points on a square with their nearest neighbors. $\endgroup$ Jan 17, 2019 at 0:11

2 Answers 2

4
$\begingroup$

We shall prove the more general result: for $n\ge2$ distinct points and any positive real $a,b$, we have $D(R):=d_1^2+\cdots+d_n^2\le 2a^2+2b^2$, where the points $P_j$ are now in an $a\times b$ rectangle $R$ and the distances $d_j$ are defined as in the OP. Let us use induction on $n$. For $n=2$, the result is obvious. Suppose now that $n>2$. Without loss of generality (wlog), $R=[0,a]\times[0,b]$. "Partition" $R$ into the four $\frac a2\times \frac b2$ rectangles $R_1:=[0,\frac a2]\times[0,\frac b2],R_2:=[\frac a2,a]\times[0,\frac b2], R_3:=[0,\frac a2]\times[\frac b2,b],R_4:=[\frac a2,a]\times[\frac b2,b]$

Let $n_1,\dots,n_4$ be the numbers of the points in the rectangles $R_1,\dots,R_4$, respectively.

Case 1: $n_1,\dots,n_4\ge2$. Then, by induction, \begin{equation} d_1^2+\cdots+d_n^2=D(R)\le D(R_1)+\dots+D(R_4)\le4[2(\tfrac a2)^2+2(\tfrac b2)^2]= 2a^2+2b^2. \end{equation}

Lemma 1. Suppose that $n_1\ge2$ and $n_2=1$, so that there is exactly one point, say $Q$, in $R_2$. Then \begin{equation} D(R_1)+d(Q,R_1)^2\le a^2+b^2, \end{equation} where $d(Q,R_1)$ is the shortest distance from $Q$ to the $n_1$ points in $R_1$.

Proof: Let $s$ be the largest abscissa of all the $n_1$ points in $R_1$, so that $0\le s\le\tfrac a2$. Let $(u,v):=Q$, so that $\tfrac a2\le u\le a$ and $0\le v\le\tfrac b2$. By symmetry, wlog $0\le v\le\tfrac b4$, and hence $d(Q,R_1)^2\le(u-s)^2+(\tfrac b2-v)^2$. So, by induction, \begin{equation} D(R_1)+d(Q,R_1)^2\le2s^2+2(\tfrac b2)^2+(u-s)^2+(\tfrac b2-v)^2\le a^2+b^2, \end{equation} which proves Lemma 1. $\Box$

Case 2: One of the $n_j$'s is $1$, and the other $n_j$'s are $\ge2$. Here wlog $n_1,n_3,n_4\ge2$ and $n_2=1$. Then, in view of Lemma 1, \begin{equation} d_1^2+\cdots+d_n^2=D(R)\le D(R_1)+d(Q,R_1)^2+D(R_3)+D(R_4)\le a^2+b^2+2[2(\tfrac a2)^2+2(\tfrac b2)^2]= 2a^2+2b^2. \end{equation}

Case 3: Two of the $n_j$'s are $1$'s, and the other $n_j$'s are $\ge2$. Here wlog $n_1,n_3\ge2$ and $n_2=n_4=1$ (the "adjacent" subcase) or $n_1,n_4\ge2$ and $n_2=n_3=1$ (the "non-adjacent" subcase). In the "adjacent" subcase, let $Q$ be the only point in $R_2$, and let $T$ be the only point in $R_4$. Then, in view of Lemma 1, \begin{equation} d_1^2+\cdots+d_n^2=D(R)\le D(R_1)+d(Q,R_1)^2+D(R_3)+d(T,R_3)^2\le a^2+b^2+a^2+b^2= 2a^2+2b^2. \end{equation} The "non-adjacent" subcase is considered quite similarly, by using an appropriate flip/symmetry.

Case 4: Three of the $n_j$'s are $1$'s, and the other $n_j$ is $\ge2$. Here wlog $n_1\ge2$ and $n_2=n_3=n_4=1$. Let $Q_2=(p,q),Q_3=(u,v),Q_4=(x,y)$ be the unique points in $R_2,R_3,R_4$, respectively, so that $p,x\in[\tfrac a2,a]$, $v,y\in[\tfrac b2,b]$, $u\in[0,\tfrac a2]$, $q\in[0,\tfrac b2]$.

Let $s$ and $t$ be, respectively, the largest abscissa and ordinate of all the $n_1$ points in $R_1$. Then
\begin{multline} D(R)\le B:=D(R_1)+d(Q_2,S)^2+d(Q_3,T)^2+\frac{\|Q_4-Q_2\|^2+\|Q_4-Q_3\|^2}2, \end{multline} where $S$ is a point (among the $n_1$ points in $R_1$) with the largest abscissa $s$, and $T$ is a point (among the $n_1$ points in $R_1$) with the largest ordinate $t$, so that $S=(s,\eta)$ for some $\eta\in[0,t]$, and $T=(\xi,t)$ for some $\xi\in[0,s]$. Of course, $d(Q_2,S)$ is a convex function of $\eta\in[0,t]$, and so, \begin{equation} d(Q_2,S)^2\le\max(d(Q_2,(s,0))^2,d(Q_2,(s,t))^2) =(p - s)^2 + \max(q^2, (q-t)^2). \end{equation} Similarly, \begin{equation} d(Q_3,T)^2\le \max(u^2, (u-s)^2) + (v - t)^2. \end{equation} Next, \begin{equation} \|Q_4-Q_2\|^2+\|Q_4-Q_3\|^2=(x - p)^2 + (y - q)^2 + (x - u)^2 + (y - v)^2, \end{equation} which is convex in $x,y$, with the maximum in $x\in[\tfrac a2,a]$ and $y\in[\tfrac b2,b]$ attained at $x=a$ and $y=b$, for any given $p\in[\tfrac a2,a]$, $v\in[\tfrac b2,b]$, $u\in[0,\tfrac a2]$, $q\in[0,\tfrac b2]$. Further, by induction, $D(R_1)\le2s^2+2t^2$. Thus, \begin{multline} D(R)\le B\le B_1:= 2 s^2+2t^2+\max \left(q^2,(q-t)^2\right)+\max \left((u-s)^2,u^2\right) \\ +(p-s)^2+(v-t)^2 \\ +\tfrac{1}{2} \left((a-p)^2+(b-q)^2\right)+\tfrac{1}{2} \left((a-u)^2+(b-v)^2\right). \end{multline} Clearly, $B_1$ is convex in $p,v$, and one can see that the maximum of $B_1$ in $p\in[\tfrac a2,a]$ and $v\in[\tfrac b2,b]$ is attained at $p=a$ and $v=b$, for any given $s\in[0,\tfrac a2]$, $t\in[0,\tfrac b2]$, $u\in[0,\tfrac a2]$, $q\in[0,\tfrac b2]$. So, \begin{equation} B_1\le B_{21}+B_{22}+B_{23}, \end{equation} where \begin{align} B_{21}&:=\max \left(q^2,(q-t)^2\right)- b q+q^2/2, \\ B_{22}&:=\max \left(u^2,(u-s)^2\right)- a u+u^2/2, \\ B_{23}&:=\tfrac{1}{2} \left(3 a^2+3 b^2-4 a s-4 b t+6 \left(s^2+t^2\right)\right). \end{align} Next, \begin{equation} B_{21}\le t^2,\quad B_{22}\le s^2 \end{equation} for $q,t\in[0,\tfrac b2]$ and $u,s\in[0,\tfrac a2]$. So, \begin{equation} D(R)\le B\le B_1\le B_{21}+B_{22}+B_{23} \le\tfrac{1}{2} (3 a^2 + 3 b^2) -4 (\tfrac a2 - s) s - 4 (\tfrac b2 - t) t \le\tfrac{1}{2} (3 a^2 + 3 b^2)\le2 a^2 + 2 b^2. \end{equation}

Case 5: This case obtains from Cases 2, 3, 4 by replacing there some of the conditions of the form $n_j=1$ by $n_j=0$. This case immediately follows from Cases 2, 3, 4, because now the nonnegative contributions of the single points will be replaced by $0$ --- with the only exception occurring when three of the $n_j$'s are $0$ and hence the remaining one of the $n_j$'s (say $n_1$) equals $n$. Indeed, in the latter exceptional subcase, we cannot use the induction, since $n_1=n\not<n$. The remedy in this case is to continue the "partitioning" of the smaller rectangles containing all the $n$ points until we no longer have such an exceptional situation. This process will stop. Indeed, if it never stops, then all the $n$ distinct points will get eventually contained in a singleton set, which is a contradiction.

So far, we have consider all the cases when at least one of the $n_j$'s is $\ge2$. Otherwise, we have $n_j\le1$ for all $j=1,\dots,4$ and hence $n\le4$. Since $n>2$, it remains to consider the following two cases.

Case 6: $n=3$. By shrinking the rectangle $R$, wlog we may assume that each side of $R$ contains at least one of the three points $P_1,P_2,P_3$. By the pigeonhole principle, at least two sides of the rectangle must share one of the three points. Also, wlog none of the points $P_1,P_2,P_3$ is in the interior of $R$. Indeed, if e.g. $P_3$ is in the interior of $R$, then we can move $P_3$ away from the line $\ell(P_1,P_2)$ (through $P_1,P_2$) in the direction perpendicular to $\ell(P_1,P_2)$ (till we hit the boundary of $R$), so that all the pairwise distances between the points $P_1,P_2,P_3$ may only increase, and then $D(R)$ may only increase.

So, wlog $P_1=(0,0),P_2=(u,b),P_3=(a,v)$ for some $u\in[0,a]$ and $v\in[0,b]$. So, \begin{equation} D(R)\le B_3:=\frac{P_1P_2^2+P_1P_3^2}2+\frac{P_2P_1^2+P_2P_3^2}2+\frac{P_3P_2^2+P_3P_1^2}2= P_1P_2^2+P_1P_3^2+P_2P_3^2, \end{equation} where $AB:=\|A-B\|$. Since $B_3$ is convex in $u,v$, its maximum in $u\in[0,a]$ and $v\in[0,b]$ is attained when $u\in\{0,a\}$ and $v\in\{0,b\}$, and this maximum is $2a^2+2b^2$, which proves Case 6.

Case 7: $n=4$. Again, by shrinking the rectangle $R$, wlog we may assume that each side of $R$ contains at least one of the four points $P_1,P_2,P_3,P_4$. Also, each of the four points is either (i) in the convex hull of the other three points or (ii) on the boundary of $R$. Indeed, otherwise wlog $P_4$ is in the interior of $R$, but not in the convex hull of $P_1,P_2,P_3$; more specifically, we may assume that the points $P_4$ and $P_3$ are to the opposite sides of the line $\ell(P_1,P_2)$. Then we can move $P_4$ away from the triangle $P_1P_2P_3$ in the direction perpendicular to the line $\ell(P_1,P_2)$ (till we hit the boundary of $R$), so that all the pairwise distances between the points $P_1,P_2,P_3,P_4$ may only increase, and then $D(R)$ may only increase. Therefore, wlog we have one the following subcases.

Subcase 7.1: $P_4$ is in the convex hull of $P_1,P_2,P_3$, so that $P_4=(1-s-t)P_1+sP_2+tP_3$ for some $s,t$ such that $s,t\ge0$ and $s+t\le1$. Also, here, as in Case 6, wlog $P_1=(0,0),P_2=(u,b),P_3=(a,v)$ for some $u\in[0,a]$ and $v\in[0,b]$. Hence \begin{equation} D(R)\le B_{71}:=P_1P_4^2+P_2P_4^2+P_3P_4^2+[(1-s-t)P_4P_1^2+s\,P_4P_2^2+t\,P_4P_3^2]\le2a^2+2b^2; \end{equation} the latter inequality follows easily by the convexity of $B_{71}$ in each of the variables $u,v,(s,t)$. Thus, Subcase 7.1 is done.

Subcase 7.2: All the four points $P_1,P_2,P_3,P_4$ are on the boundary of $R$.

Subsubcase 7.2.1: There is a bijection $\beta$ from the set of the points $\{P_1,P_2,P_3,P_4\}$ to the set of the sides of $R$ such that $P_j\in\beta(P_j)$ for all $j$. So, wlog $P_1=(s,0),P_2=(0,t),P_3=(u,b),P_4=(a,v)$ for some $s,u\in[0,a]$ and $t,v\in[0,b]$. Then, similarly to the case $n=3$,
\begin{equation} D(R)\le\frac{P_1P_2^2+P_1P_4^2}2+\frac{P_2P_1^2+P_2P_3^2}2 +\frac{P_3P_2^2+P_3P_4^2}2+\frac{P_4P_1^2+P_4P_3^2}2\le2a^2+2b^2, \end{equation} by convexity. Subcase 7.2.1 is done.

Subsubcase 7.2.2: One of the points $P_1,P_2,P_3,P_4$ (say $P_1$) is shared by two sides of the rectangle $R$. So, wlog $P_1=(0,0)$. Suppose that one of the two sides of $R$ (say $S_1$ and $S_2$) sharing the point $P_1$ contains one of the points $P_2,P_3,P_4$; let us say this side is $S_1$. Then we can move $P_1$ slightly along the side $S_2$ out of its position at $(0,0)$. In view of the continuity of $D(R)$ in $P_1$, we can thus get rid of the sharing -- provided that at least one of the two sides of $R$ sharing the point $P_1$ contains one of the points $P_2,P_3,P_4$. So, wlog $P_1=(0,0)$ and none of the two sides of $R$ sharing the point $P_1$ contains any of the points $P_2,P_3,P_4$. So, one of the sides of $R$ not sharing the point $P_1$ contains two of the points $P_2,P_3,P_4$. Therefore and by the interchangeability of the horizontal and vertical directions, wlog $P_1=(0,0),P_2=(u,b),P_3=(a,v),P_4=(a,w)$ for some $u\in[0,a]$ and $v,w\in[0,b]$ such that $v>w$. So,
\begin{equation} D(R)\le P_1P_2^2+P_2P_3^2+P_3P_4^2+P_4P_1^2\le2a^2+2b^2, \end{equation} again by convexity. Subcase 7.2.2 is done, as well as the entire proof of the bound $2a^2+2b^2$.

$\endgroup$
6
  • $\begingroup$ I have now considered more cases and simplified the proof, by introducing Lemma 1. $\endgroup$ Jan 17, 2019 at 3:13
  • $\begingroup$ It might be useful to note that there's still one case left - when 3 are 1s. $\endgroup$
    – user44191
    Jan 17, 2019 at 3:42
  • $\begingroup$ I have now added yet another case, which appears to be the most difficult one. $\endgroup$ Jan 17, 2019 at 21:33
  • $\begingroup$ One (hopefully) comparatively small step remains to finish the proof. $\endgroup$ Jan 17, 2019 at 21:59
  • $\begingroup$ The proof is finally completely done. $\endgroup$ Jan 21, 2019 at 2:06
0
$\begingroup$

You draw the discs $D_j$ of radius $d_j/2$ and center $P_j$. Then because $\|P_j-P_k\|\geq \frac{1}{2}(d_j+d_k)$ the discs are disjoint and we have $$\sum \pi\frac{d_j^2}{4}\leq 1+\mathcal{A} $$where $\mathcal{A}$ is the surface covered by the discs outside $\square_2$. So this give an elementary proof if all $d_j$ are smaller than $\sqrt{\pi}-1\approx 0,77$ since $(1+\mathcal{A})\leq (1+\max_j d_j)^2$.

$\endgroup$

Your Answer

By clicking “Post Your Answer”, you agree to our terms of service and acknowledge you have read our privacy policy.

Not the answer you're looking for? Browse other questions tagged or ask your own question.